LSAT and Law School Admissions Forum

Get expert LSAT preparation and law school admissions advice from PowerScore Test Preparation.

User avatar
 Dave Killoran
PowerScore Staff
  • PowerScore Staff
  • Posts: 5852
  • Joined: Mar 25, 2011
|
#71210
This game is discussed in our Podcast: LSAT Podcast Episode 31: The September 2019 LSAT Logic Games Section

Complete Question Explanation
(The complete setup for this game can be found here: https://forum.powerscore.com/lsat/viewtopic.php?t=31299)

The correct answer choice is (B).

When W is included in the display, then from the fourth rule G must be included in the display. Since G is included in the display, then from the third rule H cannot be included in the display. Thus, H is the correct answer.

The test makers made this question a bit easier than they had to, since in our setup discussion we made a further inference involving W and T. Since that inference is a bit tougher to see initially, one would have expected they used T as the answer here, but they did not.


Answer choice (B): This is the correct answer choice.
 cargostud
  • Posts: 17
  • Joined: Dec 23, 2019
|
#72963
This is what I came up with.

B) Is the correct answer (Honey).
Explanation: In following the 4th rule, If Wags is included in the display, Garnet must be in the second pen. Following the 3rd rule, Honey CANNOT be included since Garnet is on display.
 Adam Tyson
PowerScore Staff
  • PowerScore Staff
  • Posts: 5153
  • Joined: Apr 14, 2011
|
#72988
Absolutely correct! By going through the steps quickly you can prephrase that Honey is out, and then select answer B without delay and without having to test any other answer choices. Simply apply the rules, as they connect to one another, and you have your answer.

Another inference here would be that Taffy is also out, so if we didn't have answer B then we would have looked for that additional inference instead.

Get the most out of your LSAT Prep Plus subscription.

Analyze and track your performance with our Testing and Analytics Package.